Calculating work using vectors.

  • Thread starter Thread starter Random-Hero-
  • Start date Start date
  • Tags Tags
    Vectors Work
Click For Summary
To calculate the work done by a 25N force acting in the direction of vector (2,3,-1) while moving an object from point P(2,-3,1) to point Q(5,0,2), first determine the displacement vector by subtracting P from Q. The direction vector indicates the direction of the force, which is essential for calculating the force vector's components. The force vector is derived by multiplying the magnitude of the force (25N) by the unit vector of the direction vector. Finally, the work is computed using the dot product of the force vector and the displacement vector. Understanding these steps is crucial for accurately calculating work using vectors.
Random-Hero-
Messages
40
Reaction score
0

Homework Statement



Calculate the work done if a 25N force acting in a direction of vector (2,3,-1) moves an object from P(2,-3,1) to Q(5,0,2).


Homework Equations



Work = Force (dot) Displacement

The Attempt at a Solution



Well force is 25N. But as for displacement, do I just find the displacement of the points P and Q, but then what's the direction vector (2,3,-1) for? I'm just a bit confused on this.
 
Physics news on Phys.org
The displacement is going to be the vector Q-P. The direction vector is what direction the force is pushing on the object... you use the 25N part to determine what the magnitude of the force vector is. Then you take the dot product of that vector and Q-P
 
The direction vector gives you the direction of the 25N force vector. The force vector is just the magnitude of the force times the unit normal in the direction of the force. You should be able to calculate the unit normal of the direction vector (2,3,-1).
 
Question: A clock's minute hand has length 4 and its hour hand has length 3. What is the distance between the tips at the moment when it is increasing most rapidly?(Putnam Exam Question) Answer: Making assumption that both the hands moves at constant angular velocities, the answer is ## \sqrt{7} .## But don't you think this assumption is somewhat doubtful and wrong?

Similar threads

  • · Replies 7 ·
Replies
7
Views
715
  • · Replies 19 ·
Replies
19
Views
1K
  • · Replies 20 ·
Replies
20
Views
3K
Replies
6
Views
4K
  • · Replies 14 ·
Replies
14
Views
2K
  • · Replies 2 ·
Replies
2
Views
2K
  • · Replies 16 ·
Replies
16
Views
6K
  • · Replies 16 ·
Replies
16
Views
2K
  • · Replies 5 ·
Replies
5
Views
3K
Replies
3
Views
2K